-
Notifications
You must be signed in to change notification settings - Fork 2
/
Copy pathNumber_Series.tex
712 lines (538 loc) · 29.5 KB
/
Number_Series.tex
1
2
3
4
5
6
7
8
9
10
11
12
13
14
15
16
17
18
19
20
21
22
23
24
25
26
27
28
29
30
31
32
33
34
35
36
37
38
39
40
41
42
43
44
45
46
47
48
49
50
51
52
53
54
55
56
57
58
59
60
61
62
63
64
65
66
67
68
69
70
71
72
73
74
75
76
77
78
79
80
81
82
83
84
85
86
87
88
89
90
91
92
93
94
95
96
97
98
99
100
101
102
103
104
105
106
107
108
109
110
111
112
113
114
115
116
117
118
119
120
121
122
123
124
125
126
127
128
129
130
131
132
133
134
135
136
137
138
139
140
141
142
143
144
145
146
147
148
149
150
151
152
153
154
155
156
157
158
159
160
161
162
163
164
165
166
167
168
169
170
171
172
173
174
175
176
177
178
179
180
181
182
183
184
185
186
187
188
189
190
191
192
193
194
195
196
197
198
199
200
201
202
203
204
205
206
207
208
209
210
211
212
213
214
215
216
217
218
219
220
221
222
223
224
225
226
227
228
229
230
231
232
233
234
235
236
237
238
239
240
241
242
243
244
245
246
247
248
249
250
251
252
253
254
255
256
257
258
259
260
261
262
263
264
265
266
267
268
269
270
271
272
273
274
275
276
277
278
279
280
281
282
283
284
285
286
287
288
289
290
291
292
293
294
295
296
297
298
299
300
301
302
303
304
305
306
307
308
309
310
311
312
313
314
315
316
317
318
319
320
321
322
323
324
325
326
327
328
329
330
331
332
333
334
335
336
337
338
339
340
341
342
343
344
345
346
347
348
349
350
351
352
353
354
355
356
357
358
359
360
361
362
363
364
365
366
367
368
369
370
371
372
373
374
375
376
377
378
379
380
381
382
383
384
385
386
387
388
389
390
391
392
393
394
395
396
397
398
399
400
401
402
403
404
405
406
407
408
409
410
411
412
413
414
415
416
417
418
419
420
421
422
423
424
425
426
427
428
429
430
431
432
433
434
435
436
437
438
439
440
441
442
443
444
445
446
447
448
449
450
451
452
453
454
455
456
457
458
459
460
461
462
463
464
465
466
467
468
469
470
471
472
473
474
475
476
477
478
479
480
481
482
483
484
485
486
487
488
489
490
491
492
493
494
495
496
497
498
499
500
501
502
503
504
505
506
507
508
509
510
511
512
513
514
515
516
517
518
519
520
521
522
523
524
525
526
527
528
529
530
531
532
533
534
535
536
537
538
539
540
541
542
543
544
545
546
547
548
549
550
551
552
553
554
555
556
557
558
559
560
561
562
563
564
565
566
567
568
569
570
571
572
573
574
575
576
577
578
579
580
581
582
583
584
585
586
587
588
589
590
591
592
593
594
595
596
597
598
599
600
601
602
603
604
605
606
607
608
609
610
611
612
613
614
615
616
617
618
619
620
621
622
623
624
625
626
627
628
629
630
631
632
633
634
635
636
637
638
639
640
641
642
643
644
645
646
647
648
649
650
651
652
653
654
655
656
657
658
659
660
661
662
663
664
665
666
667
668
669
670
671
672
673
674
675
676
677
678
679
680
681
682
683
684
685
686
687
688
689
690
691
692
693
694
695
696
697
698
699
700
701
702
703
704
705
706
707
708
709
710
711
712
\chapter{数项级数}
\section{数项级数基本理论}
\begin{definition}[数项级数收敛]
给定数项级数$\sum\limits_{n = 1}^{\infty}u_n$,
定义$S_n = \sum\limits_{k= 1}^n u_n$为部分和函数,
若$\lim \limits _{n \rightarrow \infty} S_n = S$收敛,则称$\sum\limits_{n = 1}^{\infty}u_n$收敛,
否则称数项级数发散。
\end{definition}
\begin{theorem}[数项级数收敛的各种条件]
给定数项级数$\sum\limits_{n = 1}^{\infty}u_n$,其收敛的条件:
\begin{itemize}
\item 必要条件(一定能推出):$\lim \limits _{n \rightarrow \infty} u_n = 0$
\item 充要条件(Cauchy准则):$\forall \epsilon, \exists N, \forall m, m+p> N$有$|u_{m+1} + \cdots + u_{m+p}| < \epsilon$
\end{itemize}
\end{theorem}
\begin{note}
一般证明发散要么不满足必要条件,要么用Cauchy准则逆否命题
\end{note}
~
\begin{exercise}[Cauchy收敛准则]
(1)证明$\sum\limits_{n = 1}^{\infty} \frac{1}{n}$发散
(2)用Cauchy收敛准则证明$\sum\limits_{n = 1}^{\infty}\frac{1}{n^2}$收敛
\end{exercise}
\begin{proof}
(1)$\exists \epsilon_0 = 0, \forall N, m = N+1, m+p = 2N$,
则$\frac{1}{m+1} + \frac{1}{m+2} + \cdots + \frac{1}{2m} \geq \frac{1}{2} = \epsilon_0$,
根据Cauchy逆否命题可证(也可以用积分判别法证明)
(2)用$\frac{1}{n^2} < \frac{1}{n(n-1)}$,
直接求和或者用Cauchy收敛准则都行
\end{proof}
~
\begin{theorem}[子列问题]
对于数项级数$\sum\limits_{n = 1}^{\infty}u_n$,若$\lim \limits _{n \rightarrow \infty}u_n = 0$,
$S_n = \sum\limits_{k = 1}^n u_k$有一个子列$S_{np}$($p$为固定整数)收敛,
则$\sum\limits_{n = 1}^{\infty}u_n$收敛
\end{theorem}
~
\begin{exercise}[交错子列]
证明$1 + \frac{1}{2} + (\frac{1}{3} - 1) + \frac{1}{4} + \frac{1}{5} + (\frac{1}{6} - \frac{1}{2}) + \cdots = \ln 3$
\end{exercise}
\begin{proof}
$S_{3n} = \sum\limits_{k = 1}^{3n}u_{k} = (1 + \frac{1}{2} + \cdots + \frac{1}{3n}) - (1 + \frac{1}{2} + \cdots + \frac{1}{n})$,
配合 Euler 常数可知$S_{3n} = \ln 3n + \gamma - \ln n - \gamma = \ln 3$
\end{proof}
~
\begin{theorem}[去除有限项]
去掉、增加、改变级数的有限项并不改变数项级数的收敛性。
\end{theorem}
\section{正项级数}
\subsection{比较原则}
\begin{theorem}[比较原则]
$\sum\limits_{n = 1}^{\infty}u_n, \sum\limits_{n = 1}^{\infty}v_n$是两个正项级数,
\begin{itemize}
\item 非极限形式:
若$\exists N > 0$使得$\forall n > N$满足$u_n \leq v_n$,则
根据$\sum\limits_{n = 1}^{\infty}v_n$收敛推出$\sum\limits_{n = 1}^{\infty}u_n$收敛;
根据$\sum\limits_{n = 1}^{\infty}u_n$发散,则$\sum\limits_{n = 1}^{\infty}v_n$也发散
\item 极限形式:$\lim \limits _{n \rightarrow \infty} \frac{u_n}{v_n} = \lambda$。
当$0 < \lambda < \infty$,同敛散。$\lambda = 0$则$\sum\limits_{n = 1}^{\infty}v_n$敛推$\sum\limits_{n = 1}^{\infty}u_n$敛。
$\lambda = \infty$时$\sum\limits_{n = 1}^{\infty}v_n$发散推$\sum\limits_{n = 1}^{\infty}v_n$发散
\end{itemize}
\end{theorem}
\begin{note}
比较原则极限形式的$0 < \lambda < \infty$情况即可以使用等价无穷小。
但在一些特殊情况下也需要用$\lambda = 0, \lambda = \infty$的形式。
\end{note}
~
\begin{example}[常用比较对象]
$\sum\limits_{n = 0}^{\infty}\frac{1}{n^p}$在$p > 1$时收敛,在$p \leq 1$时发散
\end{example}
\begin{proof}
用积分判别法
\end{proof}
~
\begin{exercise}[使用等价无穷小]
判断敛散性:(1)$\sum\limits_{n = 1}^{\infty} \frac{1}{\sqrt{n^2 + 1}}$
(2)$\sum\limits_{n = 1}^{\infty} 2^n \sin \frac{\pi}{3^n}$
(3)重点:$\sum\limits_{n = 1}^{\infty} (\sqrt[n]{2} - 1)$
(4)重点:$\sum\limits_{n = 1}^{\infty} (a^{\frac{1}{n} } + a^{- \frac{1}{n}} - 2)(a > 1)$
\end{exercise}
\begin{solution}
(1)$\frac{1}{\sqrt{n^2 + 1}} \sim \frac{1}{n}$,发散
(2)和等比级数比,等价于$(\frac{2}{3})^n \pi$,收敛
(3)根据$2^{\frac{1}{n}} - 1 \sim \frac{1}{n} \ln 2$,发散(注意这里用的是$a^x \sim x \ln a - 1$)
(4)根据$a^{\frac{1}{n}} + a^{- \frac{1}{n}} - 2 = (a^{\frac{1}{2n}} - a^{-\frac{1}{2n}})^2 = a^{-\frac{1}{n}}(a^{\frac{1}{n}} - 1)^2 \sim \frac{\ln^2 a}{n^2}$收敛
\end{solution}
~
\begin{exercise}[含对数与指数]
(1)重点:$\sum\limits_{n = 1}^{\infty} \frac{1}{3^{\ln n}}$
(2)重点:$\sum\limits_{n = 2}^{\infty}\frac{1}{(\ln n)^{\ln n}}$
(3)$\sum\limits_{n = 3}^{\infty} \frac{1}{(\ln \ln n)^{\ln n}}$
(4)特殊:$\sum\limits_{n = 2}^{\infty}\frac{1}{(\ln n)^n}$
(5)重点:$\sum\limits_{n = 1}^{\infty}\frac{1}{n^{2n \sin \frac{1}{n}}}$
(6)重点:$\sum\limits_{n=1}^{\infty}\frac{\ln (n+1)}{e^n}$
(7)$\sum\limits_{n = 1}^{\infty}(e^{\frac{1}{n^2}} - \cos \frac{\pi}{n})$
\end{exercise}
\begin{solution}
对数需要用到恒等式$a^{\ln b} = b^{\ln a}$(两侧取对数即可证明)
(1)$\frac{1}{3^{\ln n}} = \frac{1}{n^{\ln 3}}$收敛
(2)$\frac{1}{(\ln n)^{\ln n }} = \frac{1}{n^{\ln( \ln n)}} < \frac{1}{n^2}$收敛
(3)$\frac{1}{(\ln \ln n)^{\ln n}} = \frac{1}{n^{\ln \ln \ln n}} < \frac{1}{n^2}$,收敛
(4)$\frac{1}{(\ln n)^n} < \frac{1}{2^n}$,收敛
(5)$\frac{1}{n^{2n \sin \frac{1}{n}}} < \frac{1}{n^{\frac{3}{2}}}$,因此收敛。
(6)$\lim \limits _{n \rightarrow \infty} n^2 \frac{\ln (n+1)}{e^n} \rightarrow 0$
(7)Taylor展开:$e^{\frac{1}{n^2}} - \cos \frac{\pi}{n} = 1 + \frac{1}{n^2} - 1 + \frac{\pi^2}{2n^2} + o(\frac{1}{n^2}) = (1 + \frac{\pi^2}{2})\frac{1}{n^2}$收敛
\end{solution}
\begin{note}
要灵活使用比较判别法。
有时用等价无穷小,有时直接放缩,有时乘上$n^2$(一般在分母极大),有时Taylor展开
\end{note}
~
\begin{exercise}[等价无穷小的进一步应用]
(1)$a_n > 0$,证明$(1 + a_1)(1 + a_2)\cdots (1 + a_n)$与$\sum\limits_{n = 1}^{\infty}a_n$敛散性相同
(2)$a_n = (1 - \frac{p \ln n}{n} )^n$,讨论$\sum\limits_{n = 1}^{\infty}b_n$的敛散性
(3)$f(x)$在$[-1,1]$二阶连续可微,且$\lim \limits _{x \rightarrow 0}\frac{f(x)}{x} = 0$,
证明$\sum\limits_{n = 1}^{\infty}f(\frac{1}{n})$绝对收敛
\end{exercise}
\begin{proof}
(1)取对数即$\sum\limits_{n = 1}^{\infty} \ln(1 + a_n)$,
$\lim \limits _{n \rightarrow \infty} a_n = 0$时,$\lim \limits _{n \rightarrow \infty} \frac{\ln(1 + a_n)}{a_n} = 1$,故同敛散。
若$\lim \limits _{n \rightarrow \infty} a_n \neq 0$时,$\sum\limits_{n = 1}^{\infty}a_n$发散,
显然$\lim \limits _{n \rightarrow \infty} \ln(1 + a_n) \neq 0$,故也发散。
(2)$a_n = e^{n \ln(1 - \frac{p \ln n}{n})} = e^{n[-\frac{p \ln n}{n} + o(\frac{1}{n})]} = e^{-p \ln n} \sim \frac{1}{n^p}$,
当$p > 1$时收敛,
$p \leq 1$时发散。
(3)根据$\lim \limits _{x \rightarrow 0}\frac{f(x)}{x} = 0$得到$f^{\prime}(0) = 0$,
根据Lagrange余项得到$f(\frac{1}{n}) = f^{\prime}(\xi) \frac{1}{n^2}$,因此收敛
\end{proof}
~
\begin{theorem}[比式比较判别法]
$\sum\limits_{n = 1}^{\infty}u_n , \sum\limits_{n = 1}^{\infty}v_n$为正项级数,
$\exists N, \forall n \geq N$有$\frac{u_{n+1}}{u_n} \leq \frac{v_{n+1}}{v_n}$,
则$\sum\limits_{n = 1}^{\infty}v_n$收敛可推出$\sum\limits_{n = 1}^{\infty}u_n$收敛,
$\sum\limits_{n = 1}^{\infty}u_n$发散可推出$\sum\limits_{n = 1}^{\infty}v_n$发散
\end{theorem}
\begin{proof}
根据$\frac{u_n}{u_N} = \frac{u_{N+1}}{u_N} \frac{u_{N+2}}{u_{N+1}}\cdots \frac{u_n}{u_{n-1}} \leq \frac{v_{N+1}}{v_N} \frac{v_{N+2}}{v_{N+1}} \cdots \frac{v_n}{v_{n+1}} = \frac{v_n}{v_N}$,
由此推出$u_n \leq \frac{u_N}{v_N}v_n$只相差一个常数,
因此根据一般的比较判别法可得到结论。
\end{proof}
~
\begin{exercise}[比式比较判别法基本应用]
$a_n > 0, 0 < \alpha \leq 1, \beta > 1$,证明:
(1)若$\frac{a_{n+1}}{a_n} \geq (1 - \frac{1}{n})^{\alpha}$,则$\sum\limits_{n= 1}^{\infty}a_n$发散
(2)若$\frac{a_{n+1}}{a_n} \leq (1 - \frac{1}{n})^{\beta}$,则$\sum\limits_{n = 1}^{\infty}a_n$收敛
\end{exercise}
\begin{proof}
(1)根据$\frac{a_{n+1}}{a_n} \geq (\frac{n-1}{n})^{\alpha} = \frac{\frac{1}{n^{\alpha}}}{\frac{1}{(n-1)^{\alpha}}}$,
因此根据结论发散
(2)同理
\end{proof}
\subsection{比式判别法与根式判别法}
前面比较判别法常用的比较对象是$\frac{1}{n^p}$,而根式判别法与比式判别法的比较对象是等比级数。
\begin{theorem}[比式判别法与根式判别法]
$\sum\limits_{n = 1}^{\infty}u_n$是正项级数,则
\begin{itemize}
\item 比式判别法:若$\varlimsup \limits _{n \rightarrow \infty} \frac{u_{n+1}}{u_n} = q < 1$,则$\sum\limits_{n = 1}^{\infty}u_n$收敛,若$\varliminf \limits _{n \rightarrow \infty} \frac{u_{n+1}}{u_n} > 1$则发散。
\item 根式判别法:若$\varlimsup \limits _{n \rightarrow \infty} \sqrt[n]{u_n} = l < 1$
则$\sum\limits_{n = 1}^{\infty}u_n$收敛,$\varlimsup \limits_{n \rightarrow \infty} \sqrt[n]{u_n} > 1$则发散。(双上极限)
\end{itemize}
\end{theorem}
\begin{proof}
(1)若$\varlimsup \limits_{n \rightarrow \infty}\frac{u_{n+1}}{u_n} = q < 1$,
则对$\forall r \in (q,1)$,$\exists N, \forall n > N$有$\frac{u_{n+1}}{u_n} < r = \frac{r^{n+1}}{r^n}$,
而$\sum\limits_{n = 1}^{\infty}r^n$收敛,
根据比式比较判别法可知。另一侧同理。
(2.1)若$\varlimsup \limits _{n \rightarrow \infty}\sqrt[n]{u_n} = l < 1$,
则$\forall r \in (l,1)$,$\exists N, \forall n > N$有$\sqrt[n]{u_n} < r$,
即$u_n < r^n$,由于$\sum\limits_{n = 1}^{\infty}r^n$收敛,因此$\sum\limits_{n = 1}^{\infty}u_n$收敛。
(2.2)另一侧
若$\varlimsup \limits _{n \rightarrow \infty}\sqrt[n]{u_n} = l > 1$,
则$\forall r \in (1,l)$,存在子列$\sqrt[n_k]{u_{n_k}} \geq r$,
$u_{n_k} \geq r^{n_k} \rightarrow \infty$,因此$\sum\limits_{n = 1}^{\infty}u_n$发散。
\end{proof}
\begin{note}
理论上根式判别法证明发散只需要证上极限,比比式条件更弱,因此更好用。
而且由于$\varliminf \limits_{n \rightarrow \infty}\frac{u_{n+1}}{u_n} \leq \varliminf\limits_{n \rightarrow \infty}\sqrt[n]{u_n} \leq \varlimsup \limits_{n \rightarrow \infty}\sqrt[n]{u_n} \leq \varlimsup\limits_{n \rightarrow \infty}\frac{u_{n+1}}{u_n}$可知能用比式的一定能用根式,
一定要记住$\sqrt[n]{n!} \sim \frac{n}{e}$
\end{note}
~
\begin{exercise}[使用根式判别法]
用根式判别法法证明
(1)$\sum\limits_{n = 1}^{\infty}\frac{n^2}{(2 + \frac{1}{n})^n}$
(2)$\sum\limits_{n = 1}^{\infty}\frac{n!}{n^n}$
(3)$\sum\limits_{n = 1}^{\infty}\frac{n! 3^n}{n^n}$
(4)重点:$\sum\limits_{n = 1}^{\infty}\frac{(n!)^2}{(2n)!}$
(5)重点:$\sum\limits_{n = 1}^{\infty}\frac{(2n-1)!!}{n!}$
(6)讨论$\sum\limits_{n = 1}^{\infty}\frac{x^n n!}{n^n}$的收敛性,$x \geq 0$
\end{exercise}
\begin{solution}
(1)极限为$\frac{1}{2}$,收敛
(2)极限为$\frac{n/e}{n} = \frac{1}{e}$,收敛
(3)极限为$\frac{3}{e} > 1$,发散
(4)极限为$\sqrt[n]{\frac{(n!)^2}{(2n)!}} = \frac{(\frac{n}{e})^2}{\sqrt[2n]{(\frac{2n}{e}})^2} \sim \frac{1}{4}$
(5)$\sum\limits_{n = 1}^{\infty}\frac{(2n - 1)!!}{n!} = \sum\limits_{n = 1}^{\infty}\frac{(2n)!}{n!(2n)!!} = \sum\limits_{n = 1}^{\infty}\frac{(2n)!}{n!2^n n!}$,
因此根据根式判别法等价于$2 > 1$。
(7)$\lim \limits _{n \rightarrow \infty} \sqrt[n]{\frac{x^nn!}{n^n}} = \frac{x}{e}$,
$x \in (0,e)$收敛,$x \in (e,+\infty)$发散。
当$x = e$则$\frac{a_{n+1}}{a_n} = \frac{e}{(1 + \frac{1}{n})^n} > 1$,极限不趋于$0$。
\end{solution}
~
\begin{exercise}[仅能用根式判别法]
(1)证明$\sum\limits_{n = 1}^{\infty}\frac{1}{2^{n + (-1)^n}}$收敛
(2)讨论$\sum\limits_{n = 1}^{\infty} \frac{x^n}{1 + x^{2n}}$的敛散性
\end{exercise}
\begin{proof}
(1)$\lim \limits _{n \rightarrow \infty} \sqrt[n]{\frac{1}{2^{n+(-1)^n}}} = \frac{1}{2} < 1$因此收敛
(2)分母$\lim \limits _{n \rightarrow \infty} \sqrt[n]{1 + x^{2n}} = \max \{1,x^2\}$,
因此
\begin{equation*}
\lim \limits _{n \rightarrow \infty} \sqrt[n]{u_n} = \frac{x}{\max \{1,x^2\}} =
\begin{cases}
< 1, & x \neq 1\\
=1, & x = 1
\end{cases}
\end{equation*}
因此$x \neq 1$时收敛,$x = 1$时发散。
\end{proof}
~
\begin{exercise}[根式判别法上下极限]
(1)判断$b + c + b^2 + c^2 + \cdots + b^n + c^n + \cdots$的敛散性,这里$0 < b < c < 1$
\end{exercise}
\begin{solution}
(1)$\sqrt[n]{u_n}$在偶数项为$\sqrt{c}$,奇数项为$\sqrt{b}$,
因此上极限为$\sqrt{c}$,故收敛
\end{solution}
\subsection{积分判别法和拉贝判别法}
\begin{theorem}[积分判别法与拉贝判别法]
$\sum\limits_{n = 1}^{\infty}u_n$收敛最本质的充要条件为部分和函数$S_n$有界,其推论为
\begin{itemize}
\item 拉贝判别法(常用于$\frac{a_{n+1}}{a_n}\rightarrow 1$):$\lim \limits _{n \rightarrow \infty} n(1 - \frac{u_{n+1}}{u_n}) = r > 1$,则$\sum\limits_{n = 1}^{\infty} u_n$收敛。若$r < 1$则发散,$r = 1$需要具体判定
\item 积分判别法:$f$是$[1,+\infty)$上非负减函数,则$\sum\limits_{n = 1}^{\infty}f(n)$与$\int_1^{+\infty}f(x)\mathrm{d}x$同时收敛与发散
\end{itemize}
\end{theorem}
\begin{proof}
(1)拉贝判别法:
收敛情况:
根据条件,$\exists N, \forall n > N$使得
$n \left( 1- \frac{u_{n+1}}{u_n} \right) > r$得到$\frac{u_{n+1}}{u_n} < 1 - \frac{r}{n}$,
根据伯努利不等式:
\begin{equation*}
(1 + x)^r \geq 1 + rx, x > -1, r > 1 \Rightarrow \left( 1 - \frac{1}{n} \right)^r \geq 1 - \frac{r}{n}
\end{equation*}
因此$\frac{u_{n+1}}{u_n} \leq \left( 1 - \frac{1}{n} \right)^r = \left( \frac{n-1}{n} \right)^r = \frac{\frac{1}{n^r}}{\frac{1}{(n-1)^r}}$,
显然$\sum\limits_{n = 1}^{\infty}\frac{1}{n^r}$收敛,因此$\sum\limits_{n = 1}^{\infty}u_n$收敛
发散情况:$\frac{u_{n+1}}{u_n} \geq 1 - \frac{1}{n} = \frac{n-1}{n} = \frac{\frac{1}{n}}{\frac{1}{n-1}}$,
而$\sum\limits_{n = 1}^{\infty} \frac{1}{n}$发散,因此发散。
\end{proof}
\begin{note}
拉贝判别法更多地用于比式和根式判别法都失效的情况下。
\end{note}
~
\begin{exercise}[拉贝判别法]
判断敛散性(1)$\sum\limits_{n = 1}^{\infty} \frac{n^n}{e^nn!}$
\end{exercise}
\begin{solution}
(1)显然根式判别法失效。
因此设$u_n = \frac{n^n}{e^nn!}$,使用拉贝判别法以及$(1 + \frac{1}{n})^n = e - \frac{e}{2n}$(用$(1+x)^{\frac{1}{x}}$在$0$处Taylor展开)得到
\begin{equation*}
\lim \limits _{n \rightarrow \infty} n \left( 1 - \frac{u_{n+1}}{u_n} \right) = \lim \limits _{n \rightarrow \infty} n \left[ 1 - \frac{(1 + \frac{1}{n})^n}{e} \right] = \lim \limits _{n \rightarrow \infty} n \left[ \frac{1}{2n} + o(\frac{1}{n}) \right] = \frac{1}{2} < 1
\end{equation*}
根据拉贝判别法可知发散。
\end{solution}
\subsection{放缩证明敛散性}
\begin{exercise}[两个简单放缩]
(1)$a_n \geq 0$,讨论$\sum\limits_{n = 1}^{\infty}a_n$与$\sum\limits_{n = 1}^{\infty}\sqrt{a_n a_{n+1}}$收敛性的关系
(2)设$\sum\limits_{n = 1}^{\infty}a_n^2, \sum\limits_{n = 1}^{\infty}b_n^2$收敛,
证明$\sum\limits_{n = 1}^{\infty}a_nb_n, \sum\limits_{n = 1}^{\infty}(a_n + b_n)^2$均收敛
\end{exercise}
\begin{proof}
(1)$\sqrt{a_n + b_n} \leq \frac{1}{2}(a_n + b_n)$,根据比较定理可知
(2)根据Cauchy不等式,$\sum\limits_{n = 1}^{N} a_nb_n \leq \left( \sum\limits_{n = 1}^Na_n^2 \right) \left( \sum\limits_{n = 1}^N b_n^2 \right)$,
因此收敛。
$\sum\limits_{n = 1}^N (a_n+b_n)^2 = \sum\limits_{n = 1}^N a_n^2 + 2a_nb_n + b_n^2$即可
\end{proof}
\section{任意项级数}
\subsection{交错项级数}
\begin{theorem}[Leibniz判别法]
若$a_n$单减趋于$0$,
那么交替项级数$\sum\limits_{n= 1}^{\infty}(-1)^{n-1}a_n$收敛
\end{theorem}
\begin{proof}
设$a_n$单减趋于$0$,$S_n = \sum\limits_{k = 1}^n (-1)^{k-1}a_k$,
下面考虑奇偶部分和:
\begin{equation*}
\begin{cases}
S_{2n} = (a_1 - a_2) + (a_3 - a_4) + \cdots + (a_{2n-1} - a_{2n})\\
S_{2n+1} = (a_1) - (a_2 - a_3) - (a_4 - a_5) - \cdots - (a_{2n} - a_{2n+1})
\end{cases}
\end{equation*}
显然$S_{2n}$单增,$S_{2n+1}$单减,
$a_1 - a_2 \leq S_{2n} \leq S_{2n} + a_{2n+1} = S_{2n+1} \leq a_1$,
因此$S_{2n},S_{2n+1}$均有界,根据单调有界定理可知收敛,
而奇偶子列均收敛说明$S_n$收敛。
\end{proof}
~
\begin{exercise}
讨论下面级数的条件收敛与绝对收敛性:
(1)重点:$\sum\limits_{n = 1}^{\infty} \frac{(-1)^n}{n^{p + \frac{1}{n}}}$
(2)$\sum\limits_{n = 2}^{\infty}\frac{(-1)^n}{n^p \ln^q n}$
(3)$\sum\limits_{n = 1}^{\infty}(-1)^n \sin \frac{x}{n}$
(4)$\sum\limits_{n =1 }^{\infty}(-1)^n \frac{\arctan n}{n^x}$
(5)重点:$\sum\limits_{n = 1}^{\infty} \sin (n \pi + \frac{1}{n^p})$
(6)重点:$\sum\limits_{n = 1}^{\infty} \sin (\pi \sqrt{n^2 + p^2})$
\end{exercise}
\begin{solution}
(1)$\frac{1}{n^{p + \frac{1}{n}}} \sim \frac{1}{n^p}$因此$p > 1$时绝对收敛。
条件收敛性只需要看是否单调,
考虑$1 \geq p > 0$,$f(x) = x^{p + \frac{1}{x}}$,
$f^{\prime}(x) = p x^{p-1}x^{\frac{1}{x}} + x^p x^{\frac{1}{x}} \frac{1 - \ln x}{x^2} = x^{p-1}x^{\frac{1}{x}}(p + \frac{1 - \ln x}{x}) > 0$($x$足够大时),
因此整体单减趋于$0$。
$p \leq 0$时$\lim \limits _{n \rightarrow \infty} \frac{(-1)^n}{n^{p + \frac{1}{n}}} \neq 0$,因此发散。
(2)加绝对值后可以用积分判别法,在$p > 1$时绝对收敛,$p = 1, q > 1$绝对收敛。
$p > 0$,构造$g(x) = x^p \ln^q x$,得到$g^{\prime}(x) = x^{p-1}\ln^{q-1} x(p \ln x + q) > 0$($x$足够大),
从而$p = 1, q \leq 1$时,$0 < p < 1$条件收敛
$p = 0$时$q > 0$条件收敛,$q \leq 0$发散。
$p < 0$发散。
(3)$x = 0$则绝对收敛。
$x \neq 0$时$|(-1)^n \sin \frac{x}{n}| \sim \frac{|x|}{n}$发散。
而$\sin \frac{x}{n}$在$n$足够大时单调,因此条件收敛
(4)加绝对值时等价于$\frac{1}{n^x}$,因此$x > 1$时绝对收敛。
不加绝对值时,$x < 0$一定发散,
$x > 0$时,取$g(y) = \frac{\arctan y}{y^x}$,求导发现小于$0$,因此收敛
(5)等于$\sum\limits_{n = 1}^{\infty}(-1)^n \sin \frac{1}{n^p}$,
$p > 1$绝对收敛,$0 < p \leq 1$条件收敛,
$ p \leq 0$发散。
(6)等于$\sum\limits_{n = 1}^{\infty}(-1)^n \sin(\pi \sqrt{n^2 + p^2} - n\pi) = \sum\limits_{n = 1}^{\infty}(-1)^n \sin \frac{p^2 \pi}{\sqrt{n^2 + p^2} + n}$,
因此$p = 0$时绝对收敛,
$p \neq 0$条件收敛。
\end{solution}
\begin{note}
单调性常常使用求导辅助。
\end{note}
\subsection{Dirichlet与Abel判别法}
\begin{theorem}[Dirichlet判别法]
$a_k,b_k$为两个数列,
且满足(1)$\sum\limits_{i = 1}^k a_i$有界(2)$b_k$单调趋于$0$,
则$\sum\limits_{k = 1}^{\infty}a_kb_k$收敛
\end{theorem}
\begin{theorem}[Abel判别法]
$a_k,b_k$满足(1)$\sum\limits_{i = 1}^{\infty}a_i$收敛(2)$b_k$单调有界,
则$\sum\limits_{k=1}^{\infty}a_kb_k$收敛
\end{theorem}
\begin{note}
Leibniz判别法本质只是Dirichlet和Abel判别法的一个特例
\end{note}
\begin{exercise}[常用三角结论]
(1)重点:$a_n$单减趋于$0$,$x \neq 2k\pi$,证明:$\sum\limits_{n = 1}^{\infty}a_n \cos nx, \sum\limits_{n = 1}^{\infty}a_n \sin nx$收敛
(2)重点:$x \neq k\pi$,讨论$\sum\limits_{n = 1}^{\infty}\frac{\cos nx}{n^p}, \sum\limits_{n = 1}^{\infty}\frac{\sin nx}{n^p}$的条件收敛和绝对收敛性
(3)重点:讨论$\sum\limits_{n = 1}^{\infty}(-1)^n \frac{\sin n}{n}$与$\sum\limits_{n= 1}^{\infty}(-1)^n \frac{\sin^2 n}{n}$的条件收敛与绝对收敛性
\end{exercise}
\begin{proof}
(1)根据$|\sum\limits_{k = 1}^n \cos kx| = |\frac{\sin(n + \frac{1}{2})x - \sin \frac{x}{2}}{2 \sin \frac{x}{2}}| \leq \frac{1}{|\sin \frac{x}{2}|}$,
同理$|\sum\limits_{k = 1}^{\infty}\sin kx| = |\frac{\cos \frac{x}{2} - \cos(n + \frac{1}{2})x}{2\sin \frac{x}{2}}| \leq \frac{1}{|\sin \frac{x}{2}|}$,
根据Dirichlet判别法可知。
(2)$p > 1$时$|\frac{\cos nx}{n^p}| \leq \frac{1}{n^p}$,因此绝对收敛。
$p > 0$时根据Dirichlet可知条件收敛,
$p \leq 0$时$\lim \limits _{n \rightarrow \infty} \frac{\cos nx}{n^p} \neq 0$发散。
下面证明$0 < p \leq 1$时不绝对收敛:
\begin{equation*}
\left| \frac{\cos nx}{n^p} \right| \geq
\left| \frac{\cos ^2 nx}{n^p} \right| \geq
\left| \frac{1 - \sin^2 x}{n^p} \right|
\end{equation*}
而$\sum\limits_{n = 1}^{\infty} \frac{1}{n^p}$发散,因此发散。
(3)这种双交错要把$(-1)^n$吸收进三角。
$\sin(x + n\pi) = (-1)^n \sin x, \cos(n + n\pi) = (-1)^n \cos x$,
因此$\sum\limits_{n = 1}^{\infty}(-1)^n \frac{\sin n}{n} = \sum\limits_{n = 1}^{\infty}\frac{\sin(1 + \pi)n}{n}$条件收敛(根据(1)结论)。
第二个转换为$\frac{1}{2} \sum\limits_{n = 1}^{\infty}\frac{(-1)^n}{n} - \frac{1}{2}\sum\limits_{n = 1}^{\infty}\frac{\cos(2+\pi)n}{n}$条件收敛。
绝对值发散的原因是$\sum\limits_{n = 1}^{\infty}\frac{\sin^2 n}{n} = \frac{1}{2}\sum\limits_{n = 1}^{\infty}\frac{1}{n} - \frac{1}{2} \sum\limits_{n = 1}^{\infty}\frac{\cos 2n}{n}$
\end{proof}
~
\begin{exercise}[具体例子]
讨论$\sum\limits_{n = 1}^{\infty} \frac{\cos 3n}{n} (1 + \frac{1}{n})^n$的收敛性
\end{exercise}
\begin{solution}
$\sum\limits_{n = 1}^{\infty} \frac{\cos 3n}{n}$根据前面结论收敛,
$(1 + \frac{1}{n})^n \rightarrow e$单调有界,因此由Abel可知收敛
\end{solution}
\section{数项级数理论}
\subsection{级数的加括号}
级数的加括号本质是数列收敛及其子列收敛的关系
\begin{theorem}[加括号收敛]
$\sum\limits_{n = 1}^{\infty}u_n$收敛,
则$\sum\limits_{n = 1}^{\infty}u_n$中任意加括号得到级数$\sum\limits_{n = 1}^{\infty} v_n$也收敛,
且$\sum\limits_{n = 1}^{\infty}u_n = \sum\limits_{n = 1}^{\infty}v_n$(即加括号既不改变收敛性,又不改变和)
\end{theorem}
\begin{proof}
显然$S_n = \sum\limits_{k = 1}^n u_k$收敛,
从而$S_k^{\prime} = \sum\limits_{k = 1}^mv_k$是$S_n$子列,
从而收敛且极限相同。
\end{proof}
\begin{note}
若级数发散,则加括号得到的新级数可能收敛,例如$\sum\limits_{n = 1}^{\infty}(-1)^n$。
若级数加括号以后发散,则原级数一定收敛。
\end{note}
\begin{theorem}[加括号收敛反推级数收敛]
若$\sum\limits_{n = 1}^{\infty}u_n$加括号后$\sum\limits_{n = 1}^{\infty}v_n$收敛,
\begin{itemize}
\item 若$\lim \limits _{n \rightarrow \infty} u_n = 0$且每个括号里项个数小于固定值$L$,则$\sum\limits_{n = 1}^{\infty}u_n$收敛
\item 若每个括号里符号相同,则$\sum\limits_{n = 1}^{\infty}u_n$收敛
\end{itemize}
\end{theorem}
\subsection{级数的重排}
\begin{definition}[重排]
考虑级数$\sum\limits_{n = 1}^{\infty}u_n$,
$f$是$[n] \rightarrow [n]$的映射,
则称
\begin{equation*}
\sum\limits_{n = 1}^{\infty}v_n = \sum\limits_{n = 1}^{\infty} u_{f(n)}
\end{equation*}
为$\sum\limits_{n = 1}^{\infty}u_n$的一个重排
\end{definition}
\begin{theorem}[正项级数的重排]
正项级数$\sum\limits_{n = 1}^{\infty}u_n$收敛,
则其任意重排$\sum\limits_{n = 1}^{\infty}u_{f(n)}$也收敛,
且
\begin{equation*}
\sum\limits_{n = 1}^{\infty}u_n = \sum\limits_{n = 1}^{\infty}u_{f(n)}
\end{equation*}
\end{theorem}
\begin{proof}
记$S_n = \sum\limits_{k = 1}^n u_k, S^{\prime}_n = \sum\limits_{k = 1}^n u_{f(k)}$,
记$M = \max \{f(1),\cdots, f(n)\}$,
则显然$S_n^{\prime} \leq S_{M}$,
根据$\lim \limits _{n \rightarrow \infty} S_n$存在可知$\lim \limits _{n \rightarrow \infty}S_n^{\prime}$存在,
且
\begin{equation*}
\lim \limits _{n \rightarrow \infty} S_n^{\prime} \leq \lim \limits _{n \rightarrow \infty} S_n^{\prime}
\end{equation*}
反之也可以将$S_n$看作$S_n^{\prime}$的重排,因此也满足$\lim \limits _{n \rightarrow \infty}S_n \leq \lim \limits _{n \rightarrow \infty} S_n^{\prime}$
\end{proof}
\begin{theorem}[一般项级数的重排]
$f$是重排映射,
若$\exists M \in \mathbb{Z}^+$使得$|f(n) - n| \leq M$,
则$\sum\limits_{n = 1}^{\infty} u_n$收敛当且仅当$\sum\limits_{n = 1}^{\infty} u_{f(n)}$收敛,
且
\begin{equation*}
\sum\limits_{n = 1}^{\infty }u_n = \sum\limits_{n = 1}^{\infty} u_{f(n)}
\end{equation*}
\end{theorem}
\begin{theorem}[绝对收敛级数的重排]
设$\sum\limits_{n = 1}^{\infty}u_n$绝对收敛,
则其任意重排$\sum\limits_{n = 1}^{\infty}u_{f(n)}$绝对收敛,
且
\begin{equation*}
\sum\limits_{n = 1}^{\infty}u_n = \sum\limits_{n = 1}^{\infty}u_{f(n)}
\end{equation*}
\end{theorem}
\begin{proof}
正负部分开即可
\end{proof}
\begin{theorem}[Riemann定理:条件收敛级数的重排]
设数项级数$\sum\limits_{n = 1}^{\infty}u_n$条件收敛,
则对$\forall S \in \mathbb{R} \cup \{\pm \infty\}$,
存在重排$\sum\limits_{n = 1}^{\infty}v_n$收敛到$S$
\end{theorem}
\subsection{正部与负部}
对于任意数列$u_n$,考虑非负数列$p_n = \frac{|u_n| + u_n}{2}, q_n = \frac{|u_n| - u_n}{2}$,
分别为$u_n$的正部与负部,下面研究它们之间的关系。
\begin{theorem}[正负部]
若$\sum\limits_{n = 1}^{\infty}u_n$绝对收敛,则$\sum\limits_{n = 1}^{\infty}p_n, \sum\limits_{n = 1}^{\infty}q_n$均收敛。
若$\sum\limits_{n = 1}^{\infty}u_n$条件收敛,则$\sum\limits_{k = 1}^np_k, \sum\limits_{k = 1}^nq_k$发散到正无穷,且$\lim \limits _{n \rightarrow \infty} \frac{\sum\limits_{k = 1}^n p_k}{\sum\limits_{k = 1}^n q_k} = 1$
\end{theorem}
\begin{proof}
(1)$0 \leq p_n, q_n \leq |u_n|$,因此根据比较原则可发现$\sum\limits_{n = 1}^{\infty}p_n,q_n$都收敛,
且极限可拆开
(2)$\sum\limits_{n = 1}^{\infty}u_n$条件收敛,则$\sum\limits_{n = 1}^{\infty}(p_n - q_n)$收敛,
但$\sum\limits_{n = 1}^{\infty}(p_n + q_n) = +\infty$,
假设$\sum\limits_{n = 1}^{\infty}p_n$收敛,则$\sum\limits_{n = 1}^{\infty}q_n$收敛,这与$\sum\limits_{n = 1}^{\infty}(p_n + q_n)$发散矛盾。
\end{proof}
\begin{exercise}
$a_n$单减,$a_n \geq 0$,$\sum\limits_{n = 1}^{\infty}a_n$发散,
证明$\lim \limits _{n \rightarrow \infty} \frac{a_2 + a_4 + \cdots + a_{2n}}{a_1 + a_3 + \cdots + a_{2n-1}} = 1$
\end{exercise}
\begin{proof}
极限小于等于$1$显然。
而$\frac{a_2 + \cdots + a_{2n}}{a_1 + \cdots + a_{2n-1}} \geq \frac{a_3 + a_5 + \cdots + a_{2n-1}}{a_1 + a_3 + \cdots + a_{2n-1}} = 1 - \frac{a_1}{a_1 + \cdots + a_{2n-1}}$,
根据单减以及发散可知$2(a_1 + a_3 + \cdots + a_{2n-1}) \geq a_1 + a_2 + \cdots + a_{2n} \rightarrow \infty$,
因此大于等于$1$。
\end{proof}
\subsection{级数的乘积}
\subsection{级数的相互控制}
\begin{theorem}[级数相互控制]
设$a_n \leq c_n \leq b_n$则
\begin{itemize}
\item 数列$a_n,b_n$收敛不能得出$c_n$收敛
\item 级数$\sum\limits_{n = 1}^{\infty}a_n, \sum\limits_{n = 1}^{\infty}b_n$收敛可得出$\sum\limits_{n = 1}^{\infty}c_n$收敛
\end{itemize}
\end{theorem}
\begin{proof}
(1)例如:$a_n = -2 - \frac{1}{n}, b_n = 2 + \frac{1}{n}, c_n = (-1)^n$
(2)此时$0 \leq c_n - a_n \leq b_n - a_n$,
显然$\sum\limits_{n = 1}^{\infty}b_n - a_n$收敛,
根据比较原则得到$\sum\limits_{n = 1}^{\infty}c_n - a_n$收敛,因此得到$\sum\limits_{n = 1}^{\infty} c_n$收敛。
\end{proof}
\begin{corollary}[控制是相互的]
若$a_n \leq b_n \leq a_{n+1}$,则显然推出$b_{n-1} \leq a_n \leq b_n$,
因此$\sum\limits_{n = 1}^{\infty}a_n, \sum\limits_{n = 1}^{\infty}b_n$同敛散。
\end{corollary}
~
\begin{exercise}[相互控制练习]
(1)重点:$a_n$每一项都非零,$\lim \limits _{n \rightarrow \infty} a_n = a \neq 0$,证明$\sum\limits_{n = 1}^{\infty}|a_{n+1} - a_n|, \sum\limits_{n = 1}^{\infty} \left| \frac{1}{a_{n+1}} - \frac{1}{a_n} \right|$敛散性相同
(2)$a_n$是单调减的正数列,证明$\sum\limits_{n = 1}^{\infty}a_n$与$\sum\limits_{m = 1}^{\infty}2^m a_{2^m}$同敛散
\end{exercise}
\begin{proof}
(1)根据极限保号性,$\exists N, \forall n > N$有$\frac{1}{2}a \leq a_n \leq \frac{3}{2}a$,
而$\left| \frac{1}{a_{n+1}} - \frac{1}{a_n} \right| = \left| \frac{a_n - a_{n+1}}{a_na_{n+1}} \right|$
因此
\begin{equation*}
\frac{4}{9a^2} |a_{n+1} - a_n| \leq \frac{|a_{n+1} - a_n|}{|a_na_{n+1}|} \leq \frac{4}{a^2}|a_{n+1} - a_n|
\end{equation*}
从而相互控制,同敛散
(2)根据$a_n$单调减可得
\begin{equation*}
2^{k-1}a_{2^k} \leq a_{2^{k-1}+1} + a_{2^{k-1}+2} + \cdots + a_{2^k} \leq 2^{k-1}a_{2^{k-1}}
\end{equation*}
对$k$求和得到
\begin{equation*}
\frac{1}{2} \sum\limits_{k = 1}^n 2^k a_{2^k} \leq a_1 + a_2 + \cdots + a_{2^n} \leq a_1 + \sum\limits_{k = 0}^{n-1}2^k a_{2^k}
\end{equation*}
这说明两者部分和等价,因此同敛散。
\end{proof}
~
\begin{exercise}[比较原则对于一般项级数不成立的反例]
(1)$a_n,b_n$满足$\lim \limits _{n \rightarrow \infty} \frac{a_n}{b_n} = l > 0$,
此时显然$\sum\limits_{n = 1}^{\infty}|a_n|, \sum\limits_{n = 1}^{\infty}|b_n|$敛散性相同,
但举例说明$\sum\limits_{n = 1}^{\infty}a_n, \sum\limits_{n = 1}^{\infty}b_n$敛散性可以不同。
(2)$\lim \limits _{n \rightarrow \infty} \frac{a_n}{b_n} = 0$,根据$\sum\limits_{n = 1}^{\infty}b_n$绝对收敛可推出$\sum\limits_{n = 1}^{\infty}a_n$绝对收敛,
举例$\sum\limits_{n = 1}^{\infty}b_n$收敛推不出$\sum\limits_{n = 1}^{\infty}a_n$收敛
(3)举例说明$\sum\limits_{n = 1}^{\infty}a_n$收敛,$\sum\limits_{n = 1}^{\infty}a_n^2$不一定收敛
\end{exercise}
\begin{solution}
(1)例如$a_n = \frac{(-1)^n}{\sqrt{n}}, b_n = \frac{(-1)^n}{\sqrt{n} } + \frac{1}{n}$,
显然$\lim \limits _{n \rightarrow \infty} \frac{a_n}{b_n} = 1$,但$a_n$收敛,$b_n$发散
(2)比如$a_n = \frac{1}{n}, b_n = \frac{(-1)^n}{\sqrt{n}}$
(3)例如$a_n = \frac{(-1)^n}{\sqrt{n}}$
\end{solution}